Question and Answers Forum

All Questions      Topic List

Mechanics Questions

Previous in All Question      Next in All Question      

Previous in Mechanics      Next in Mechanics      

Question Number 154549 by peter frank last updated on 19/Sep/21

A particle is projected  with velocity  2(√(gh))   so that  it just clears two  walls of equal heigh(h)  in the  t_1  and t_(2 )  respectively.The two  walls are at a distance of  2h  from  each other.If time passing   between the two walls is 2(√(h/g))  show that (i) angle projected 60^°                 (ii)t_1 +t_2 =2(√((3h)/g))

$$\mathrm{A}\:\mathrm{particle}\:\mathrm{is}\:\mathrm{projected}\:\:\mathrm{with}\:\mathrm{velocity} \\ $$$$\mathrm{2}\sqrt{\mathrm{gh}}\:\:\:\mathrm{so}\:\mathrm{that}\:\:\mathrm{it}\:\mathrm{just}\:\mathrm{clears}\:\mathrm{two} \\ $$$$\mathrm{walls}\:\mathrm{of}\:\mathrm{equal}\:\mathrm{heigh}\left(\mathrm{h}\right)\:\:\mathrm{in}\:\mathrm{the} \\ $$$$\mathrm{t}_{\mathrm{1}} \:\mathrm{and}\:\mathrm{t}_{\mathrm{2}\:} \:\mathrm{respectively}.\mathrm{The}\:\mathrm{two} \\ $$$$\mathrm{walls}\:\mathrm{are}\:\mathrm{at}\:\mathrm{a}\:\mathrm{distance}\:\mathrm{of}\:\:\mathrm{2h}\:\:\mathrm{from} \\ $$$$\mathrm{each}\:\mathrm{other}.\mathrm{If}\:\mathrm{time}\:\mathrm{passing}\: \\ $$$$\mathrm{between}\:\mathrm{the}\:\mathrm{two}\:\mathrm{walls}\:\mathrm{is}\:\mathrm{2}\sqrt{\frac{\mathrm{h}}{\mathrm{g}}} \\ $$$$\mathrm{show}\:\mathrm{that}\:\left(\mathrm{i}\right)\:\mathrm{angle}\:\mathrm{projected}\:\mathrm{60}^{°} \\ $$$$\:\:\:\:\:\:\:\:\:\:\:\:\:\:\left(\mathrm{ii}\right)\mathrm{t}_{\mathrm{1}} +\mathrm{t}_{\mathrm{2}} =\mathrm{2}\sqrt{\frac{\mathrm{3h}}{\mathrm{g}}} \\ $$$$ \\ $$

Commented by Tawa11 last updated on 21/Sep/21

nice sir

$$\mathrm{nice}\:\mathrm{sir} \\ $$

Answered by ARUNG_Brandon_MBU last updated on 19/Sep/21

(i)For horizontal motion between both walls;   x=v_x t. But x=2h, v_x =2(√(gh))cosϑ, t=2(√(h/g))  ⇒2h=(2(√(gh))cosϑ)(2(√(h/g)))  ⇒(1/2)=cosϑ⇒ϑ=60°

$$\left({i}\right)\mathrm{For}\:\mathrm{horizontal}\:\mathrm{motion}\:\mathrm{between}\:\mathrm{both}\:\mathrm{walls}; \\ $$$$\:{x}={v}_{{x}} {t}.\:\mathrm{But}\:{x}=\mathrm{2}{h},\:{v}_{{x}} =\mathrm{2}\sqrt{{gh}}\mathrm{cos}\vartheta,\:{t}=\mathrm{2}\sqrt{\frac{{h}}{{g}}} \\ $$$$\Rightarrow\mathrm{2}{h}=\left(\mathrm{2}\sqrt{{gh}}\mathrm{cos}\vartheta\right)\left(\mathrm{2}\sqrt{\frac{{h}}{{g}}}\right) \\ $$$$\Rightarrow\frac{\mathrm{1}}{\mathrm{2}}=\mathrm{cos}\vartheta\Rightarrow\vartheta=\mathrm{60}° \\ $$

Commented by peter frank last updated on 20/Sep/21

thanks

$$\mathrm{thanks} \\ $$

Answered by ARUNG_Brandon_MBU last updated on 19/Sep/21

(ii) For vertical motion:  y=ut+(1/2)at^2 , y=h, u=2(√(gh))sin60°=(√(3gh)), a=−g  ⇒h=(√(3gh))t−(1/2)gt^2 ⇒gt^2 −2(√(3gh))t+h=0  t=((2(√(3gh))±(√(12gh−4gh)))/(2g))=((2(√(3gh))±2(√(2gh)))/(2g))  t_1 =(√((3h)/g))+(√((2h)/g)), t_2 =(√((3h)/g))−(√((2h)/g))  t_1 +t_2 =2(√((3h)/g))

$$\left({ii}\right)\:\mathrm{For}\:\mathrm{vertical}\:\mathrm{motion}: \\ $$$${y}={ut}+\frac{\mathrm{1}}{\mathrm{2}}{at}^{\mathrm{2}} ,\:{y}={h},\:{u}=\mathrm{2}\sqrt{{gh}}\mathrm{sin60}°=\sqrt{\mathrm{3}{gh}},\:{a}=−{g} \\ $$$$\Rightarrow{h}=\sqrt{\mathrm{3}{gh}}{t}−\frac{\mathrm{1}}{\mathrm{2}}{gt}^{\mathrm{2}} \Rightarrow{gt}^{\mathrm{2}} −\mathrm{2}\sqrt{\mathrm{3}{gh}}{t}+{h}=\mathrm{0} \\ $$$${t}=\frac{\mathrm{2}\sqrt{\mathrm{3}{gh}}\pm\sqrt{\mathrm{12}{gh}−\mathrm{4}{gh}}}{\mathrm{2}{g}}=\frac{\mathrm{2}\sqrt{\mathrm{3}{gh}}\pm\mathrm{2}\sqrt{\mathrm{2}{gh}}}{\mathrm{2}{g}} \\ $$$${t}_{\mathrm{1}} =\sqrt{\frac{\mathrm{3}{h}}{{g}}}+\sqrt{\frac{\mathrm{2}{h}}{{g}}},\:{t}_{\mathrm{2}} =\sqrt{\frac{\mathrm{3}{h}}{{g}}}−\sqrt{\frac{\mathrm{2}{h}}{{g}}} \\ $$$${t}_{\mathrm{1}} +{t}_{\mathrm{2}} =\mathrm{2}\sqrt{\frac{\mathrm{3}{h}}{{g}}} \\ $$

Answered by peter frank last updated on 20/Sep/21

Answered by peter frank last updated on 20/Sep/21

x−direction  x_1 =v_o cos θt_1   x_2 =v_o cos θt_2   v_o =2(√(gh))  x_2 −x_1 =2h      (t_2 −t_1 )=2(√(h/g))  v_o cos θ(t_2 −t_1 )=2h  cosθ=((2h)/(v_o (t_2 −t_1 )))  =(1/2)     θ=  cos^(−1) (1/2)=60^°   also  y−direction  y=v_o sin θt_1 −(1/2)gt_1 ^2   y=v_o sin θt_2 −(1/2)gt_2 ^2   v_o sin θt_1 −(1/2)gt_1 ^2 =v_o sin θt_2 −(1/2)gt_2 ^2   2(√(gh)) sin θ(t_2 −t_1 )=(1/2)g(t_2 ^2 −t_1 ^2 )  2(√(gh)) sin θ=(1/2)g(t_2 +t_1 )   [θ=60]  t_2 +t_1 =((4(√(gh)))/g).((√3)/2)=  t_2 +t_1 =2(√(((3h)/g) ))

$$\mathrm{x}−\mathrm{direction} \\ $$$$\mathrm{x}_{\mathrm{1}} =\mathrm{v}_{\mathrm{o}} \mathrm{cos}\:\theta\mathrm{t}_{\mathrm{1}} \\ $$$$\mathrm{x}_{\mathrm{2}} =\mathrm{v}_{\mathrm{o}} \mathrm{cos}\:\theta\mathrm{t}_{\mathrm{2}} \\ $$$$\mathrm{v}_{\mathrm{o}} =\mathrm{2}\sqrt{\mathrm{gh}} \\ $$$$\mathrm{x}_{\mathrm{2}} −\mathrm{x}_{\mathrm{1}} =\mathrm{2h}\:\:\:\:\:\:\left(\mathrm{t}_{\mathrm{2}} −\mathrm{t}_{\mathrm{1}} \right)=\mathrm{2}\sqrt{\frac{\mathrm{h}}{\mathrm{g}}} \\ $$$$\mathrm{v}_{\mathrm{o}} \mathrm{cos}\:\theta\left(\mathrm{t}_{\mathrm{2}} −\mathrm{t}_{\mathrm{1}} \right)=\mathrm{2h} \\ $$$$\mathrm{cos}\theta=\frac{\mathrm{2h}}{\mathrm{v}_{\mathrm{o}} \left(\mathrm{t}_{\mathrm{2}} −\mathrm{t}_{\mathrm{1}} \right)}\:\:=\frac{\mathrm{1}}{\mathrm{2}}\:\:\: \\ $$$$\theta=\:\:\mathrm{cos}^{−\mathrm{1}} \frac{\mathrm{1}}{\mathrm{2}}=\mathrm{60}^{°} \\ $$$$\mathrm{also}\:\:\mathrm{y}−\mathrm{direction} \\ $$$$\mathrm{y}=\mathrm{v}_{\mathrm{o}} \mathrm{sin}\:\theta\mathrm{t}_{\mathrm{1}} −\frac{\mathrm{1}}{\mathrm{2}}\mathrm{gt}_{\mathrm{1}} ^{\mathrm{2}} \\ $$$$\mathrm{y}=\mathrm{v}_{\mathrm{o}} \mathrm{sin}\:\theta\mathrm{t}_{\mathrm{2}} −\frac{\mathrm{1}}{\mathrm{2}}\mathrm{gt}_{\mathrm{2}} ^{\mathrm{2}} \\ $$$$\mathrm{v}_{\mathrm{o}} \mathrm{sin}\:\theta\mathrm{t}_{\mathrm{1}} −\frac{\mathrm{1}}{\mathrm{2}}\mathrm{gt}_{\mathrm{1}} ^{\mathrm{2}} =\mathrm{v}_{\mathrm{o}} \mathrm{sin}\:\theta\mathrm{t}_{\mathrm{2}} −\frac{\mathrm{1}}{\mathrm{2}}\mathrm{gt}_{\mathrm{2}} ^{\mathrm{2}} \\ $$$$\mathrm{2}\sqrt{\mathrm{gh}}\:\mathrm{sin}\:\theta\left(\mathrm{t}_{\mathrm{2}} −\mathrm{t}_{\mathrm{1}} \right)=\frac{\mathrm{1}}{\mathrm{2}}\mathrm{g}\left(\mathrm{t}_{\mathrm{2}} ^{\mathrm{2}} −\mathrm{t}_{\mathrm{1}} ^{\mathrm{2}} \right) \\ $$$$\mathrm{2}\sqrt{\mathrm{gh}}\:\mathrm{sin}\:\theta=\frac{\mathrm{1}}{\mathrm{2}}\mathrm{g}\left(\mathrm{t}_{\mathrm{2}} +\mathrm{t}_{\mathrm{1}} \right)\:\:\:\left[\theta=\mathrm{60}\right] \\ $$$$\mathrm{t}_{\mathrm{2}} +\mathrm{t}_{\mathrm{1}} =\frac{\mathrm{4}\sqrt{\mathrm{gh}}}{\mathrm{g}}.\frac{\sqrt{\mathrm{3}}}{\mathrm{2}}= \\ $$$$\mathrm{t}_{\mathrm{2}} +\mathrm{t}_{\mathrm{1}} =\mathrm{2}\sqrt{\frac{\mathrm{3h}}{\mathrm{g}}\:}\: \\ $$$$ \\ $$$$ \\ $$

Terms of Service

Privacy Policy

Contact: info@tinkutara.com